Determining which answer most helps to resolve the paradox - LSAT Logical Reasoning

Card 0 of 18

Question

After replacing her old air conditioner with a new, energy-efficient unit, Paula’s electric bills increased.

Each of the following, if true, contributes to an explanation of the increase mentioned above EXCEPT:

Answer

The correct answer here is the one that does not explain the apparent paradox. That the new unit uses a smaller share of total electricity is consistent with its being more energy efficient. It does not explain how her electric bills could have gone up despite her installation of the more efficient unit; all other answers provide possible resolutions of such apparent paradox.

Compare your answer with the correct one above

Question

TV networks believe that football broadcasts provide greater opportunity for profit than any other sport. This belief persists despite the fact that professional baseball leagues have increased their revenue by fifty percent over the last ten years while professional football leagues have increased revenue by only fifteen percent over that time.

Which of the following statements, if true, would most help to reconcile the TV networks' beliefs with the statistics cited?

Answer

The TV networks' conclusions may be justified even though baseball leagues have increased revenues more than football leagues. The revenue from TV broadcasting is of more direct relevance to the revenue of TV networks.

Compare your answer with the correct one above

Question

Recent evidence has conclusively shown that cholesterol levels do not correlate with human lifespan. Despite this new evidence, doctors still advise patients with high cholesterol to take medication and engage in physical activity to reduce cholesterol levels.

Which of the following, if true, would most help resolve this discrepancy?

Answer

The correct answer helps explain why doctors would advise their patients to reduce their cholestrol levels even if it would not increase their longevity.

Compare your answer with the correct one above

Question

Under the modern model for music distribution, musicians sell the right to broadcast their music to companies who give listeners affordable access to large quantities of music. Some musicians who have been selling their music for many years are displeased with the new model and have seen decreased revenue. Yet, musicians as a whole are making more money under the new model than they were previously.

Which of the following, if true, would most help to reconcile the apparent conflict?

Answer

The paradox in the passage is that some musicians generate less revenue now than they did under the old model while, on the whole, musicians earn more revenue than previously. If there are more musicians than under the old model, revenues could increase for the industry as a whole while decreasing for certain musicians. This is not the only information that could explain the paradox, but it does so better than any of the other answers given here.

Compare your answer with the correct one above

Question

GDP (Gross Domestic Product) is one of the primary ways that economists determine the health of an economy. Unfortunately, GDP is very difficult to calculate and many economists realize that official GDP reports are inaccurate. Despite this, many economists feel that they can accurately gauge the health of an economy.

Which one of the following, if true, most helps resolve the apparent contradiction in the paragraph above?

Answer

We are looking for an answer that helps explain why economists feel like they can accurately gauge the health of the economy despite the fact that they think their primary means of doing so may have flaws. The correct answer helps resolve this paradox by showing that there could be another metric that economists use to help gauge the health of the economy. The other answers either are irrelevant or make the paradox harder to explain.

Compare your answer with the correct one above

Question

Much of the recent scholarship surrounding counter-terrorism studies has fallen under heavy criticism for its ambiguous use of labels, its lack of interviews with actual terrorists, and its inability to create reproducible results. Despite these shortcomings, Dr. Jane Waxman, a Professor of terrorism studies, has concluded that these counter-terrorism studies are very useful.

Which of the following, if true, would help explain the apparent discrepancy between the Professor’s conclusion and the problems surrounding recent scholarship on counter-terrorism?

Answer

We are looking for an answer that helps explain why Dr. Waxman finds the existing counter-terrorism scholarship useful in spite of its weaknesses. The correct answer does this by showing that the weak scholarship can be useful to future scholars. The rest of the answers do not help resolve the paradox.

Compare your answer with the correct one above

Question

Over the past ten years, insurance premiums have increased, resulting in a large decrease in insurance enrollment across the country. Insurance company revenues, however, have progressively increased in each of the ten years during this period, and industry analysts predict further increases in years to come.

Which one of the following, if true, offers the best explanation for the situation described above?

Answer

Here, the author describes two simultaneous trends that, at first blush, seem to contradict one another. Insurance premiums have increased, which have led to a decrease in insurance enrollment. This first part is logical. However, interestingly, revenues for insurance companies have continued to rise despite the decreased enrollment. The correct answer choice must provide additional revenue that compensates for the decrease in revenue from decreased enrollment. Only the answer choice: “The decrease in the number of people enrolling for health insurance over the past ten years has been more than offset by the increases in insurance premiums” provides this increased revenue that would explain this phenomenon.

Compare your answer with the correct one above

Question

During his second semester of college, Tom enrolled in tutoring sessions to help him improve his grades. However, his GPA for his second semester was worse than his GPA for his first semester.

Each of the following, if true, helps to explain why Tom's GPA worsened, EXCEPT:

Answer

The correct answer is:

The tutoring sessions took more time than Tom had realized they would take.

This does not resolve the paradox, because even if the tutoring sessions took more time than Tom realized they would required, this does not necessarily mean that he did not have the time to utilize them. All the other answer choices do in fact resolve the paradox because they shed light on why Tom's GPA would have worsened.

Compare your answer with the correct one above

Question

Recent polls of admissions officers at colleges have shown that when evaluating student applications, volunteering is valued less than taking on leadership roles in student organizations. However, despite having this information, some high school counselors encourage students to spend time volunteering over taking on leadership roles in student organizations.

Which of the following, if true, resolves this discrepancy?

Answer

The correct answer is:

Some high school counselors believe that the personal development that can result form volunteering is more important than the competitive edge that a leadership role can earn a student applying for college.

In order to justify why a high school counselor might favor volunteering over student leadership roles, it is important to show that there is an incentive more powerful than helping a student gain an edge for getting into college. This is the only answer choice that provides such an incentive. Therefore, it is the correct choice.

Compare your answer with the correct one above

Question

It is time to face the fact that public libraries throughout the country should be converted to fitness facilities. The country's citizens are progressively becoming less healthy and must have access to fitness facilities in order to improve health. Meanwhile, public libraries take up space but do not provide value to the public. The best course of action is clear.

The author of the passage is most likely to agree with which of the following?

Answer

The author does not make a judgment about the historic value provided by public libraries, leaving that question open. The author does state, however, that citizens "must have access to fitness facilities in order to improve health." Thus, the author apparently holds the view that access to fitness facilities is necessary for improved health.

Compare your answer with the correct one above

Question

Smoking cigarettes continues to be a common practice despite countless studies demonstrating that the habit causes deadly diseases. The government has attempted to reduce the number of smokers with advertising campaigns that explain the consequences of smoking, but the campaigns have been ineffective because the habit continues to be as prevalent as it was previous to the campaigns. The government should try one or more of the many alternatives to advertising. For instance, the government could raise the tax on cigarettes to the extent that smoking is prohibitively expensive.

Which of the following, if true, most weakens the argument?

Answer

A key premise of the argument is that the government's advertising campaign has been unsuccessful because smoking is as prevalent as it was prior to the campaign. The campaign may still be successful, however, if campaigns are generally successful only after many years. The new information would undermine a fundamental premise of the argument.

Compare your answer with the correct one above

Question

Eye surgery has become a viable alternative to wearing glasses. There have been questions about long term effectiveness for many years, but the doubt seems to be disappearing. Few people have reported negative effects even years after surgery and many people say their improved vision has lasted. We should see the demand for eye surgery increasing drastically as a result of doubts being dispelled. Yet, the number of people obtaining eye surgeries remains relatively constant.

Which of the following most helps to explain the paradox?

Answer

If most people do not have eye surgery because of the cost, then dispelling concerns about effectiveness may not lead to more eye surgeries.

Compare your answer with the correct one above

Question

Two years ago, the state legislature significantly increased the fines for distracted driving offenses in an attempt to reduce traffic accidents. The legislature also made such offenses more severe with regard to possible driver license suspensions, and publicized the new penalties on highway signs and television commercials. Yet recent studies show that since the changes, the number of tickets for distracted driving has actually increased statewide.

Which one of the following, if true, most helps to resolve the apparent discrepancy in the passage above?

Answer

The correct answer choice resolves the discrepancy by explaining that an overall population increase could increase the total number of tickets despite the deterring effects of the legislative change. Note that it is the total number of tickets, not the average tickets per person, that has increased. The only other answer choice that comes close to explaining the discrepancy is that some drivers are unaware of the changes. However, this answer choice is weak, as it only applies to “some” drivers; more importantly, it does not explain why the total number of tickets has increased rather than stayed the same. The other answer choices are not useful in explaining the discrepancy.

Compare your answer with the correct one above

Question

The owner of a local convenience store wants to increase store revenue. Soft-serve ice cream machines provide a cheap snack for customers on a hot day and also produce ice cream at a low production cost. The owner of the local convenience store installs a soft-serve ice cream machine to increase revenue. After the soft-serve ice cream machine is installed, the revenue of the store goes down.

Which of the following best explains the drop in revenue?

Answer

If the customers start buying cheap snacks instead of expensive snacks, that could explain the drop in revanue. If the icecream brings in more customers and if those customers spend less there is the possibility that it could increase revenue. Maintenance costs are expenses not revenue. Finally, be careful to avoid answers that are not relevant to the question or that do not make sense, such as the ‘relative impact’ answer.

Compare your answer with the correct one above

Question

A computer technician notices several bugs in a computer indicating that the computer has one or more viruses. The computer technician scans the computer for viruses. After discovering several viruses in the system, the technician runs anti-virus software. When the software is finished, the technician confirms that all viruses have been deleted but that the bugs in the computer are still present.

Which of the following statements does NOT resolve the paradox described above?

Answer

Whether or not several viruses were initially detected by the anti-virus software does not matter because it is clearly stated that all viruses were deleted. In fact, even if several viruses were not initially detected, they would still have been deleted by the anti-virus software.

Compare your answer with the correct one above

Question

A child sells lemonade by the road. The child put ice in the lemonade. When lemonade has ice in it, lemonade is cool to drink and refreshing. On hot days, pedestrians and motorists will only drink cool and refreshing beverages. But the child sells more lemonade on cold days than on hot days.

Which of the following best resolves the above paradox?

Answer

If the ice melts the lemonade will not be made cool and refreshing. Because pedestrians and motorists will only drink beverages that are cool and refreshing on hot days, the child will not sell any lemonade on hot days because the lemonade will not be cool and refreshing.

Compare your answer with the correct one above

Question

Apartment buildings are better at withstanding tornadoes than mobile homes are, because they are anchored more solidly to the ground by their foundations. In a recent tornado, however, an apartment building was destroyed, while the mobile homes in the adjacent lots were not damaged.

Which of the following, if true, best explains the aftermath of the tornado described above?

Answer

The relative number of each type of structure in a given area does not explain why one was damaged and another was not (the question did not say that MORE apartments were damaged than mobile homes). The question also did not say anything about the occupants inside the homes, and whether or not occupants are in the home does not determine whether or not the homes themselves are damaged. The fact that all buildings are somewhat vulnerable does not explain the difference in damage. The only answer choice that explains the unexpected difference is the one that implies that the structure that is typically supposed to be stronger was somehow weakened in a previous event.

Compare your answer with the correct one above

Question

There were few people who actually understood what the professor was lecturing about, but none of his students challenged the statements. Instead, they all decided to trudge through their reading and assignments in hopes those would make the subject clear.

Which of the following statements best resolves the paradox presented above?

Answer

The professor's lecture is stated as being incomprehensible to his students, but that they are hopeful that the subject matter can be made clear through the textbook. This paradox, that the students think they can understand the material although not understanding the specific lecture, can be resolved by the fact that the textbook will explain the material in a much clearer manner than the professor.

Compare your answer with the correct one above

Tap the card to reveal the answer